Вывод закона ома для полной цепи: Закон Ома для полной (замкнутой) цепи

Содержание

Закон Ома для полной (замкнутой) цепи

Закон Ома для полной цепи определяет значение тока в реальной цепи, который зависит не только от сопротивления нагрузки, но и от сопротивления самого источника тока. Другое название этого закона — закон Ома для замкнутой цепи. Рассмотрим смысл закона Ома для полной цепи более подробно.

Потребители электрического тока (например, электрические лампы) вместе с источником тока образуют замкнутую электрическую цепь. На рисунке 1 показана замкнутая электрическая цепь, состоящая из автомобильного аккумулятора и лампочки.

Рисунок 1. Замкнутая цепь, поясняющея закон Ома для полной цепи.

Ток, проходящий через лампочку, проходит также и через источник тока. Следовательно, проходя по цепи, ток кроме сопротивления проводника встретит еще и то сопротивление, которое ему будет оказывать сам источник тока (сопротивле­ние электролита между пластинами и сопротивление пограничных слоев электролита и пластин).

Следовательно, общее сопротивление замкнутой цепи будет складываться из сопротивления лампочки и сопротивления источника тока.

Сопротивление нагрузки, присоединенной к источнику тока, принято называть внешним сопротивлением, а со­противление самого источника тока — внутренним со­противлением. Внутреннее сопротивление обозначается буквой r.

Если по цепи, изображенной на рисунке 1, протекает ток I, то для поддержания этого тока во внешней цепи согласно за­кону Ома между ее концами должна существовать раз­ность потенциалов, равная I*R. Но этот же ток I протекает и по внутренней цепи. Следовательно, для поддержания тока во внутренней цепи, также необходимо существование разности потенциалов между концами сопротивления r. Эта разность потенциалов па закону Ома должна быть равна I*r.

Поэтому для поддержания тока в цепи электродвижущая сила (ЭДС) аккумулятора должна иметь величину:

E=I*r+I*R

Эта формула показывает, что электродвижущая сила в цепи равна сумме внешнего и внутреннего падений напряжения. Вынося I за скобки, получим:

E=I(r+R)

или

I=E/(r+R)

Две последние формулы выражают закона Ома для полной цепи.

Закон Ома для полной замкнутой цепи формулируется так: сила тока в замкнутой цепи прямо пропорциональ­на ЭДС в цепи и обратно пропорциональ­на общему сопротивлению цепи.

Под общим со­противлением подразумевается сумма внешнего и внутреннего сопротивлений.

ПОНРАВИЛАСЬ СТАТЬЯ? ПОДЕЛИСЬ С ДРУЗЬЯМИ В СОЦИАЛЬНЫХ СЕТЯХ!

Похожие материалы:

Добавить комментарий

Закон Ома для полной цепи | Физика. Закон, формула, лекция, шпаргалка, шпора, доклад, ГДЗ, решебник, конспект, кратко

Рис. 5.19. Внутренняя и внешняя части электрической цепи

Рассмотрим замкнутую электрическую цепь, состоящую из двух частей: собственно источника с электродвижущей силой Ɛ и внутренним сопротивлением r и внешней части цепи — проводника с сопротивлением R (рис. 5.19).

Закон Ома для полной цепи устанав­ливает зависимость силы тока в замкнутой цепи I от электродвижущей силы источника Ɛ и полного сопротивления цепи R + r. Эту зависимость можно установить на основании закона сохранения энергии и закона Джоу­ля-Ленца. Если через поперечное сечение проводника за время Δt заряженными час­тицами переносится заряд Δq, то работа сторонних сил

Aст. = ƐΔq = ƐIΔt.

Если в цепи электрическая энергия прев­ращается лишь в тепловую, то по закону со­хранения энергии Аст. = Q и общее коли­чество теплоты, выделяющееся в замкнутой цепи, равно сумме количеств теплоты, вы­деляющихся во внешней и внутренней час­тях цепи

Q = I2RΔt + I2rΔt.

Если

Aст. = Q = (Ɛ / R + r) • IΔt,

то

ƐIΔt = I2RΔt + I2rΔt.

Итак,

Ɛ = IR + Ir

и

I = Ɛ / (R + r),

что и выражает закон Ома для полной цепи.

Закон Ома для полной цепи. Сила тока в замкнутой цепи измеряется отно­шением электродвижущей силы источника тока, имеющегося в этой цепи, к полному ее сопротивлению.

Из сказанного выше можно сделать вы­вод, что

закон Ома для полной цепи являет­ся одним из выражений закона сохранения энергии.

Во многих случаях для характеристики источников тока недостаточно использовать лишь ЭДС. Пусть, например, необходимо установить, ток какой максимальной силы может дать определенный источник тока. Если исходить из закона Ома для полной цепи

I = Ɛ / (R + r), Материал с сайта http://worldofschool. ru

то очевидно, что максимальной сила тока в цепи будет тогда, когда внешнее сопротивление цепи R стремится к нулю — это короткое замыкание в цепи. При этом ток

короткого замыкания имеет силу Imax = Ɛ / r, поскольку Ɛ и r изменить для данного источника мы не можем, они яв­ляются характеристиками источника.

Если представить, что сопротивление вне­шней части цепи стремится к бесконеч­ности (цепь становится разомкнутой), то напряжение на полюсах источника тока IR стремится к электродвижущей силе, то есть:

электродвижущая сила источника тока равна напряжению на полюсах разомкнутого источ­ника.

На этой странице материал по темам:
  • Закон ома шпора

  • Реферат на тему закон ома для полной цепи

  • Закон ома для полной цепи эссе

  • Закон ома полная лекция по физике

  • Запишите формулу закона ома для полной цепи.

Вопросы по этому материалу:
  • Как определяется работа сторонних сил?

  • Сформулируйте закон Ома для полной цепи.

  • Запишите формулу закона Ома для полной цепи.

  • Что такое ток короткого замыкания?

  • Как можно опре­делить ток короткого замыкания?

  • Как связаны между собой максимально возможное напряжение на полюсах источника и электродвижущая сила источника?

Изучение закона ома для полной цепи вывод. Лабораторная работа

Цель работы:

Углубление знаний о законе Ома для участков цепи и о законе Ома для полной цепи. Применения правил Кирхгофа для расчета цепей постоянного тока.

Оборудование : учебно-лабораторный стенд «Законы постоянного тока», мультиметр, три-четыре резистора с известными сопротивлениями, два гальванических элемента разных типов, соединительные провода.

Введение

Постановка задачи о расчете цепи постоянного тока: «Зная величины действующих в цепи э.д.с., внутренние сопротивления источников тока и сопротивления всех элементов цепи, рассчитать силы токов на каждом участке цепи и падение напряжения на каждом элементе».

При решении этой задачи используются:

закон Ома для участка цепи

I – сила тока, U – напряжение на участке цепи, R – сопротивление участка;

закон Ома для полной цепи

I – сила тока, e э.д.с. источника тока, R – сопротивление внешней цепи,

r – внутреннее сопротивление источника тока.

Непосредственный расчет разветвленных цепей, содержащих несколько замкнутых контуров и несколько источников тока, производится с помощью двух правил Кихгофа.

Любая точка в разветвленной цепи, в которой сходится не менее трех проводников с током, называется узлом . При этом ток, входящий в узел, считается положительным, а ток, выходящий из узла, — отрицательным.

Первое правило Кирхгофа : алгебраическая сила токов, сходящихся в узле, равна нулю:

Второе правило Кирхгофа : в любом замкнутом контуре, произвольно выбранном в разветвленной цепи, алгебраическая сумма произведений сил токов на сопротивления соответствующих участков этого контура равна алгебраической сумме э.д.с., встречающихся в контуре:

(4)

Описание стенда «Законы постоянного тока»

В работе используется стенд, состоящий из двух источников тока (гальванических элементов), набора из четырёх резисторов с известными сопротивлениями, мультиметра и набора соединительных проводов.

1. При сборке электрических цепей необходимо обеспечить хороший контакт в каждом соединении.

2. Соединительные провода закручиваются под клеммы по часовой стрелке .

3. При измерении сил токов и напряжений щупы мультиметра должны быть плотно прижаты к клеммам.

4. Измерения производятся при кратковременном замыкании цепи кнопкой.

5. Не следует длительное время оставлять цепь в собранном состоянии.

Прежде всего, изучите правила измерений с помощью универсального электроизмерительного прибора – мультиметра.

Измерение, обработка и представление результатов измерений

Задание 1.

Э.д.с. источника тока можно с достаточно большой степенью точности измерить непосредственно с помощью вольтметра. Но при этом следует иметь в виду, что при этом измеряемое напряжение меньше истинного значения э.д.с. на величину падения напряжения на самом источнике тока.

, (5)

где U – показания вольтметра.

Разница между истинным значением э.д.с. и измеренным напряжением при этом равна:

. (6)

При этом относительная погрешность измерения э.д.с. равна:

(7)

Обычно сопротивление источника тока (гальванического элемента) равно несколько Ом (например, 1Ом ). Если даже сопротивление вольтметра мало (например, 100 Ом ), то и в этом случае погрешность прямого измерения э. д.с. составляет всего » 1%. Хороший вольтметр, в том числе используемый в мультиметре, имеет сопротивление порядка 10 6 Ом . Ясно, что при использовании такого вольтметра можно считать, что показание вольтметра практически равно измеряемой э.д.с источника тока.

1. Подготовьте мультиметр к измерению постоянного напряжения до 2 В .

2. Не вынимая гальванические элементы из креплений, измерьте и запишите их э.д.с. с точностью до сотых долей вольта.

3. Э.д.с. величина всегда положительная. Соблюдайте полярность при подключении мультиметра к источникам тока. Красный щуп мультиметра присоединяется к «+» источника тока.

Задание 2.

Внутреннее сопротивление источника тока можно вычислить с помощью закона Ома:

1. Подготовьте мультиметр для измерения силы постоянного тока до 10(20) А .

2. Составьте электрическую цепь из последовательно соединенного источника тока, резистора (одного из набора) и амперметра.

3. Измерьте силу тока в цепи.

4. Рассчитайте и запишите величину внутреннего сопротивления источника.

5. Аналогичные измерения проделайте для другого элемента.

Задание 3. Расчёт электрической цепи постоянного тока

1. Соберите электрическую цепь по схеме, предложенной преподавателем (схемы 1-7).

2. Зачертите схему в отчет по работе и укажите номиналы выбранных резисторов.

3. С помощью правил Кирхгофа рассчитайте силы токов во всех ветвях цепи. Вычислите падения напряжений на каждом резисторе.

4. С помощью мультимета измерьте силу тока в доступном для измерения месте. Измерьте падение напряжения на каждом резисторе.

5. В выводе сравните измеренные и расчетные значения и укажите причины возможных расхождений.

Задание 4. Соединение источников тока в батареи

1. Источники тока могут соединятся в батареи двумя основными способами: параллельно и последовательно. Если источники соединяются последовательно, то их э.д.с. и внутренние сопротивления складываются:

При параллельном соединении одинаковых источников тока общая э.д.с. батареи равна э.д.с. одного источника, а внутреннее сопротивление батареи в n раз меньше внутреннего сопротивления одного источника тока:

(10)

Соберите цепи по схемам 8, 9, в которых реализуются обе схемы соединения. Рассчитайте и измерьте силу тока в цепи при этих соединениях. В выводе сравните расчетные и измеренные значения.

Отчет по лабораторной работе № 3

Изучение применения закона Ома для расчета цепей постоянного тока

выполненной учащимся школы «Поиск»

…………………………………………………………………………………

«…….»………….. 200….

Задание 1. Определение э.д.с. источников тока

Первый источник тока e 1 = ……… В

Второй источник тока e 2 = ……… В

Задание 2. Измерение внутреннего сопротивления источников тока

Первый источник тока

R = ……… Ом, I = ……… А, r 1 = ……… Ом

Второй источник тока

R = ……… Ом, I = ……… А, r 2 = ……… Ом

Таблица 1

Вывод: ………………………………………………………………………………………………………………………………………………………………………………………………………………………………………………………………………………………………………………………

При проектировании и ремонте схем различного назначения обязательно учитывается закон Ома для полной цепи. Поэтому тем, кто собирается этим заниматься, для лучшего понимания процессов этот закон надо знать. Законы Ома разделяют на две категории:

  • для отдельного участка электрической цепи;
  • для полной замкнутой цепи.

В обоих случаях учитывается внутреннее сопротивление в структуре источника питания. В вычислительных расчетах используют закон Ома для замкнутой цепи и другие определения.

Простейшая схема с источником ЭДС

Чтобы понять закон Ома для полной цепи, для наглядности изучения рассматривается самая простая схема с минимальным количеством элементов, ЭДС и активной резистивной нагрузки. Можно прибавить в комплект соединительные провода. Для питания идеально подходит автомобильный аккумулятор 12В, он рассматривается как источник ЭДС со своим сопротивлением в элементах конструкции.

Роль нагрузки играет обычная лампа накаливания с вольфрамовой спиралью, которая имеет сопротивление в несколько десятков Ом. Данная нагрузка преобразует электрическую энергию в тепловую. Всего несколько процентов расходуются на излучение потока света. При расчете таких схем применяют закон Ома для замкнутой цепи.

Принцип пропорциональности

Экспериментальными исследованиями в процессе измерений величин при разных значениях параметров полной цепи:

  • Силы тока – I А;
  • Суммы сопротивлений батареи и нагрузки – R+r измеряют в омах;
  • ЭДС – источник тока, обозначают как Е. измеряется в вольтах

было замечено, что сила тока имеет прямо пропорциональную зависимость относительно ЭДС и обратную пропорциональную зависимость относительно суммы сопротивлений, которые замыкаются последовательно в контуре цепи. Алгебраически это сформулируем следующим образом:

Рассматриваемый пример схемы с замкнутым контуром цепи – с одним источником питания и одним внешним элементом сопротивления нагрузки в виде лампы со спиралью накаливания. При расчете сложных схем с несколькими контурами и множеством элементов нагрузки применяют закон Ома для всей цепи и другие правила. В частности надо знать законы Киргофа, понимать, что такое двухполюсники, четырехполюсники, отводящие узлы и отдельные ветви. Это требует детального рассмотрения в отдельной статье, раньше этот курс ТЭРЦ (теория электро- радиотехнических цепей) в институтах учили не менее двух лет. Поэтому ограничиваемся простым определением только для полной электрической цепи.

Особенности сопротивлений в источниках питания

Важно! Если сопротивление спирали на лампе мы видим на схеме и в реальной конструкции, то внутреннего сопротивления в конструкции гальванической батарейки, или аккумулятора, не видно. В реальной жизни, даже если разобрать аккумулятор, найти сопротивление невозможно, оно не существует как отдельная деталь, иногда его отображают на схемах.

Внутреннее сопротивление создается на молекулярном уровне. Токопроводящие материалы аккумулятора или другого источника питания генератора с выпрямителем тока не обладают 100% проводимостью. Всегда присутствуют элементы с частицами диэлектрика или металлов другой проводимости, это создает потери тока и напряжения в батарее. На аккумуляторах и батарейках нагляднее всего отображается влияние сопротивления элементов конструкции на величину напряжения и тока на выходе. Способность источника выдавать максимальный ток определяет чистота состава токопроводящих элементов и электролита. Чем чище материалы, тем меньше значение r, источник ЭДС выдает больший ток. И, наоборот, при наличии примесей ток меньше, r увеличивается.

В нашем примере аккумулятор имеет ЭДС 12В, к нему подключается лампочка, способная потреблять мощность 21 Вт, в этом режиме спираль лампы раскаляется до максимально допустимого накала. Формулировка проходящего через нее тока записывается как:

I = P\U = 21 Вт / 12В = 1,75 А.

При этом спираль лампы горит в половину накала, выясним причину этого явления. Для расчетов сопротивления общей нагрузки (R + r ) применяют законы Ома для отдельных участков цепей и принципы пропорциональности:

(R + r) = 12\ 1,75 = 6,85 Ом.

Возникает вопрос, как выделить из суммы сопротивлений величину r. Допускается вариант – измерить мультиметром сопротивление спирали лампы, отнять его от общего и получить значение r – ЭДС. Этот способ будет не точен – при нагревании спирали сопротивление значительно изменяет свою величину. Очевидно, что лампа не потребляет заявленной в ее характеристиках мощности. Ясно, что напряжение и ток для накаливания спирали малы. Для выяснения причины измерим падение напряжения на аккумуляторе при подключенной нагрузке, к примеру, оно будет 8 Вольт. Предположим, что сопротивление спирали рассчитывается с использованием принципов пропорциональности:

U/ I = 12В/1,75А = 6,85 Ом.

При падении напряжения сопротивление лампы остается постоянным, в этом случае:

  • I = U/R = 8В/6,85 Ом = 1,16 А при требуемом 1.75А;
  • Потери по току = (1,75 -1.16) = 0,59А;
  • По напряжению = 12В – 8В = 4В.

Потребляемая мощность будет Р = UxI = 8В х 1.16А = 9,28 Вт вместо положенных 21 Вт. Выясняем, куда уходит энергия. За пределы замкнутого контура не может, остаются только провода и конструкция источника ЭДС.

Сопротивление ЭДС – r можно вычислить, используя потерянные величины напряжения и тока:

r = 4В/0.59А = 6,7 Ом.

Получается внутреннее сопротивление источника питания «сжирает» половину выделяемой энергии на себя, и это, конечно, не нормально.

Такое бывает в старых отработавших свой срок или бракованных аккумуляторах. Сейчас производители стараются следить за качеством и чистотой применяемых токоведущих материалов, чтобы снизить потери. Для того чтобы в нагрузку отдавалась максимальная мощность, технологии изготовления источников ЭДС контролируют, чтобы величина не превышала 0,25 Ом.

Зная закон Ома для замкнутой цепи, используя постулаты пропорциональности, можно легко вычислить необходимые параметры для электрических цепей для определения неисправных элементов или проектирования новых схем различного назначения.

Видео

Закон Ома для полной цепи – эмпирический (полученный из эксперимента) закон, который устанавливает связь между силой тока, электродвижущей силой (ЭДС) и внешним и внутренним сопротивлением в цепи.

При проведении реальных исследований электрических характеристик цепей с постоянным током необходимо учитывать сопротивление самого источника тока. Таким образом в физике осуществляется переход от идеального источника тока к реальному источнику тока, у которого есть свое сопротивление (см. рис. 1).

Рис. 1. Изображение идеального и реального источников тока

Рассмотрение источника тока с собственным сопротивлением обязывает использовать закон Ома для полной цепи.

Сформулируем закона Ома для полной цепи так (см. рис. 2): сила тока в полной цепи прямо пропорциональна ЭДС и обратно пропорциональна полному сопротивлению цепи, где под полным сопротивлением понимается сумма внешних и внутренних сопротивлений.

Рис. 2. Схема закона Ома для полной цепи.


  • R – внешнее сопротивление [Ом];
  • r – сопротивление источника ЭДС (внутреннее) [Ом];
  • I – сила тока [А];
  • ε– ЭДС источника тока [В].

Рассмотрим некоторые задачи на данную тему. Задачи на закон Ома для полной цепи, как правило, дают ученикам 10 класса, чтобы они могли лучше усвоить указанную тему.

I. Определите силу тока в цепи с лампочкой, сопротивлением 2,4 Ом и источником тока, ЭДС которого равно 10 В, а внутреннее сопротивление 0,1 Ом.

По определению закона Ома для полной цепи, сила тока равна:

II. Определить внутреннее сопротивление источника тока с ЭДС 52 В. Если известно, что при подключении этого источника тока к цепи с сопротивлением 10 Ом амперметр показывает значение 5 А.

Запишем закон Ома для полной цепи и выразим из него внутреннее сопротивление:

III. Однажды школьник спросил у учителя по физике: «Почему батарейка садится?» Как грамотно ответить на данный вопрос?

Мы уже знаем, что реальный источник обладает собственным сопротивлением, которое обусловлено либо сопротивлением растворов электролитов для гальванических элементов и аккумуляторов, либо сопротивлением проводников для генераторов. Согласно закону Ома для полной цепи:

следовательно, ток в цепи может уменьшаться либо из-за уменьшения ЭДС, либо из-за повышения внутреннего сопротивления. Значение ЭДС у аккумулятора почти постоянный. Следовательно, ток в цепи понижается за счет повышения внутреннего сопротивления. Итак, «батарейка» садится, так как её внутреннее сопротивление увеличивается.

Тема: «Изучение закона Ома для участка цепи»

Цель работы : установить на опыте зависимость силы тока от напряжения и сопротивления.

Оборудование : амперметр лабораторный, вольтметр лабораторный, источник питания, набор из трёх резисторов сопротивлениями 1 Ом, 2 Ом, 4 Ом, реостат, ключ замыкания тока, соединительные провода.

Ход работы.

Краткие теоритические сведения

Электрический ток — упорядоченное движение заряженных частиц

Количественной мерой электрического тока служит сила тока I

Сила тока — скалярная физическая величина, равная отношению заряда q, переносимого через поперечное сечение проводника за интервал времени t, к этому интервалу времени:

В Международной системе единиц СИ сила тока измеряется в амперах [А] .

Прибор для измерения силы тока Амперметр. Включается в цепь последовательно

Напряжение – это физическая величина, характеризующая действие электрического поля на заряженные частицы, численно равно работе электрического поля по перемещению заряда из точки с потенциалом φ 1 в точку с потенциалом φ 2

U 12 = φ 1 – φ 2

U – напряжение

A работа тока

q электрический заряд

Единица напряжения – Вольт [В]

Прибор для измерения напряжения – Вольтметр. Подключается в цепь параллельно тому участку цепи, на котором измеряется разность потенциалов.

На схемах электрических цепей амперметр обозначается .

Величина, характеризующая противодействие электрическому току в проводнике, которое обусловлено внутренним строением проводника и хаотическим движением его частиц, называется электрическим сопротивлением проводника.

Электрическое сопротивление проводника зависит от размеров и формы проводника и от материала , из которого изготовлен проводник .

S – площадь поперечного сечения проводника

l длина проводника

ρ – удельное сопротивление проводника

В СИ единицей электрического сопротивления проводников служит ом [Ом].

Графическая зависимость силы тока I от напряжения U вольт-амперная характеристика

Закон Ома для однородного участка цепи : сила тока в проводнике прямо пропорциональна приложенному напряжению и обратно пропорциональна сопротивлению проводника.

Назван в честь его первооткрывателя Георга Ома .

Практическая часть

1. Для выполнения работы соберите электрическую цепь из источника тока, амперметра, реостата, проволочного резистора сопротивлением 2 Ом и ключа. Параллельно проволочному резистору присоедините вольтметр (см. схему).

2. Опыт 1.

Таблица 1 . Сопротивление участка 2 Ом

3.

4. Опыт 2 .

Таблица 2.

5.

6. Ответьте на контрольные вопросы.

Контрольные вопросы

1. Что такое электрический ток?

2. Дайте определение силы тока. Как обозначается? По какой формуле находится?

3. Какова единица измерения силы тока?

4. Каким прибором измеряется сила тока? Как он включается в электрическую цепь?

5. Дайте определение напряжения. Как обозначается? По какой формуле находится?

6. Какова единица измерения напряжения?

7. Каким прибором измеряется напряжение? Как он включается в электрическую цепь?

8. Дайте определение сопротивления. Как обозначается? По какой формуле находится?

9. Какова единица измерения сопротивления?

10. Сформулируйте закон Ома для участка цепи.

Вариант выполнения измерений.

Опыт 1. Исследование зависимости силы тока от напряжения на данном участке цепи . Включите ток. При помощи реостата доведите напряжение на зажимах проволочного резистора до 1 В, затем до 2 В и до 3 В. Каждый раз при этом измеряйте силу тока и результаты записывайте в табл. 1.

Таблица 1 . Сопротивление участка 2 Ом

По данным опытов постройте график зависимости силы тока от напряжения. Сделайте вывод.

Опыт 2. Исследование зависимости силы тока от сопротивления участка цепи при постоянном напряжении на его концах . Включите в цепь по той же схеме проволочный резистор сначала сопротивлением 1 Ом, затем 2 Ом и 4 Ом. При помощи реостата устанавливайте на концах участка каждый раз одно и то же напряжение, например, 2 В. Измеряйте при этом силу тока, результаты записывайте в табл 2.

Таблица 2. Постоянное напряжение на участке 2 В

По данным опытов постройте график зависимости силы тока от сопротивления. Сделайте вывод.

Презентация: «Лабораторная работа: «Изучение закона Ома для участка цепи» .

{edocs}fizpr/lr7f.pptx,800,600{/edocs}

Лабораторная работа №10. «Изучение закона Ома для полной цепи – 3 способ». Цель работы: изучить закон Ома для полной цепи. Задачи работы:  определение ЭДС и внутреннего сопротивления источника постоянного тока по его вольтамперной характеристике;  исследование графической зависимости мощности, выделяющейся во внешней цепи от величины силы электрического тока P  f I  . Оборудование: источник постоянного тока, амперметр, вольтметр, соединительные провода, ключ, реостат. Теория и метод выполнения работы: Закон I  Rr Ома для полной цепи I  Rr . Преобразуем    I  R  r   I  R  I  r  U  I  r    U  I  r  U    I  r . выражение Следовательно, зависимость напряжения на выходе источника постоянного тока от величины силы тока (вольтамперная характеристика) имеет вид (см. рис. 1): рис. 1 Анализ вольт-амперной характеристики источника постоянного тока: 1) для т.C: I=0, тогда U    0  r   2) для т.D: U=0, тогда 0    I  r    I  r  I  3) tg  U   r I I к.з   I к.з r Выражение для мощности, выделяющейся во внешней электрической цепи имеет вид P  I  U  I    I  r   I    I 2  r . Поэтому графическая зависимость P  f I  представляет собой параболу, ветви которой направлены вниз (см. рис. 2). рис. 2 Анализ графической зависимости P  f I  (см. рис. 3): рис. 3 1) для т.B: P=0, тогда 0  I   I 2  r  0    I  r  I   r  I к. з. , т.е. абсцисса т.B соответствует току короткого замыкания; 2) т.к. парабола является симметричной, то абсцисса т.А составляет половину тока короткого замыкания I  3) т.к. в т.А I  I к. з.   , а ордината – соответствует максимальному значению мощности; 2 2r  Rr и I  2r , то после преобразований получаем R=r – условие, при котором мощность выделяющаяся во внешней цепи с источником постоянного тока принимает максимальное значение; 2     r  4) максимальное значение мощности P  I 2  R   .  4r 2r 2 Ход работы: 1. Подключить вольтметр к клеммам источника постоянного тока (см. рис. 4). Напряжение, показанное вольтметром принять за величину ЭДС источника постоянного тока и считать как эталонное для данной лабораторной работы. Результат записать в виде: (U±U) В. Абсолютную погрешность принять равной цене деления вольтметра. рис. 4 2. Собрать экспериментальную установку по схеме, приведённой на рисунке 5: рис. 5 3. Провести серию из 5-10 экспериментов, при плавном перемещении ползунка реостата, результаты измерений заносить в таблицу: Сила тока Напряжение I U А В 4. По полученным экспериментальным данным построить вольт-амперную характеристику источника постоянного тока. 5. Определить возможное значение ЭДС источника постоянного тока и тока короткого замыкания. 6. Применить методику графической обработки экспериментальных данных и вычислений для расчёта внутреннего сопротивления источника постоянного тока. 7. Результаты вычислений представить в виде:  ЭДС источника постоянного тока: (ср±ср) В;  внутреннее сопротивление источника постоянного тока: r=(rср±rср) Ом. 8. Построить графическую зависимость U  f I  в Microsoft Excel, используя мастер диаграмм с добавлением линии тренда и указанием уравнения прямой. По основным параметрам уравнения определить возможное значение ЭДС источника постоянного тока, тока короткого замыкания и внутреннее сопротивление. 9. На числовых осях указать интервал значений ЭДС, внутреннего сопротивления источника постоянного тока и тока короткого замыкания, полученных различными методами определения. 10. Исследовать мощность, выделяющуюся во внешней цепи от величины силы электрического тока. Для этого заполнить таблицу и построить графическую зависимость P  f I  : Сила тока Мощность I P А Вт 11. По построенному графику определить максимальное значение мощности, ток короткого замыкания, внутреннее сопротивление источника тока и ЭДС. 12. Возможен вариант построения графической зависимости P  f I  в Microsoft Excel, используя мастер диаграмм с добавлением полиномиальной линии тренда со степенью 2, пересечением кривой с осью OY (P) в начале координат и указанием уравнения на диаграмме. По основным параметрам уравнения определить максимальное значение мощности, ток короткого замыкания, внутреннее сопротивление источника тока и ЭДС. 13. Сформулировать общий вывод по работе.

Георг Ом. Закон Ома для полной цепи

1. Георг Ом

Да, электричество – мой
задушевный друг,
Согреет, развлечет,
прибавит света.
Опыты, проведенные Омом показали, что сила
тока, напряжение и сопротивление – величины,
связанные между собой.
движущиеся
заряженные частицы
Ампер
Вольт
Электрический ток создают
Единица силы тока
Единица напряжения
Единица сопротивления
Ом
Формула Закона Ома для участка цепи
I=U/R
Сила тока измеряется по формуле
I = q/ t
Прибор для измерения силы тока
Амперметр
Прибор для измерения напряжения
Вольтметр
Прибор, сопротивление которого
можно регулировать
Реостат
последовательно
Амперметр включается в цепь
Формула нахождения сопротивления
За направление тока принято
направление движения
R=ρl/S
положительно заряженных
частиц
При последовательном соединении
проводников общее сопротивление цепи равно
При параллельном соединении проводников
сила тока в цепи…
При параллельном соединении
проводников напряжение в цепи…
С изменением напряжения или силы
тока в цепи сопротивление…
Сумме всех
сопротивлений
Равна сумме
токов
Одинаково на
каждом
проводнике
Не меняется

4. Актуализация знаний.

1. Почему раньше удлинитель исправно работал, а
Актуализация знаний.
тут вдруг загорелся?
2. Какое явление произошло?
3. Какой закон необходимо исследовать для
теоретического объяснения данного явления?

6. Вывод 1:Закон Ома для участка цепи:

сила тока в участке цепи прямо
пропорциональна напряжению
на концах этого участка и
обратно пропорциональна его
сопротивлению.

7. Вольт-амперная характеристика проводника

График, выражающий зависимость
силы тока от напряжения, называется
вольт-амперной характеристикой
проводника.

9. Вывод 2:Закон Ома для полной цепи:

• Закон Ома для участка цепи
рассматривает только данный участок
цепи, а закон Ома для полной цепи
рассматривает полное сопротивление всей
цепи.
• Оба закона Ома показывают зависимость
силы тока от сопротивления – чем больше
сопротивление, тем меньше сила тока и
наоборот.

10. Закон Ома для полной цепи

Я брал куски цилиндрической проволоки произвольной
длины из различных материалов и помещал их
поочередно в цепь…
Георг Ом
…открытие Ома было скептически воспринято в научных
кругах. Это отразилось и на развитии науки – скажем, законы
распределения токов в разветвленных цепях были выведены
Г. Кирхгофом лишь двадцать лет спустя, — и на научной
карьере Ома
Вопрос
1. Какие величины
связывает закон Ома?
2. Как формулируется
закон Ома?
3. Напишите формулу
закон Ома
4. Напишите единицы
измерения
5. Вывод
Закон Ома для
участка цепи
Закон Ома для
полной цепи
Любые неэлектростатические силы, действующие на заряженные
частицы, принято называть сторонними силами. Т.о. на заряды
внутри источника, помимо кулоновских, действуют сторонние силы
и осуществляют перенос заряженных частиц против кулоновских.


+
А


е
е

Fст

В
Силы электростатического
происхождения не могут
создать и поддерживать на
концах проводника
постоянную разность
потенциалов
(электростатические силы
– консервативные силы)
происхождения, способные поддерживать разность
потенциалов на концах проводника

14. Характеристики источника тока

15. Роль источника тока

Чтобы электрический ток в проводнике не
прекращался, необходимо использовать
устройство, которое переносило бы заряды
от одного тела к другому в направлении,
противоположном
тому,
в
котором
переносятся заряды электрическим полем. В
качестве такого устройства используют
источник тока.

16. Источники электрического тока

Источник тока — это устройство, в котором происходит преобразование
какого-либо вида энергии в электрическую энергию.
Существуют различные виды источников тока:
Механический источник тока
— механическая энергия преобразуется в электрическую энергию.
К ним относятся : электрофорная машина (диски машины приводятся во
вращение в противоположных направлениях. В результате трения щеток
о
диски
на
кондукторах
машины
накапливаются
заряды
противоположного знака), динамо-машина, генераторы.
Тепловой источник тока
— внутренняя энергия преобразуется в электрическую энергию.
Например, термоэлемент — две проволоки из разных металлов
необходимо спаять с одного края, затем нагреть место спая, тогда
между другими концами этих проволок появится напряжение.
Применяются
в
термодатчиках
и
на
геотермальных
электростанциях.
Световой источник тока
— энергия света преобразуется в электрическую энергию.
Например, фотоэлемент — при освещении некоторых полупроводников
световая энергия превращается в электрическую. Из фотоэлементов
составлены солнечные батареи.
Применяются в солнечных батареях, световых датчиках, калькуляторах,
видеокамерах.
Химический источник тока
— в результате химических реакций внутренняя энергия преобразуется в
электрическую.
Например, гальванический элемент — в цинковый сосуд вставлен угольный
стержень. Стержень помещен в полотняный мешочек, наполненный смесью
оксида марганца с углем. В элементе используют клейстер из муки на
растворе нашатыря. При взаимодействии нашатыря с цинком, цинк
приобретает отрицательный заряд, а угольный стержень — положительный
заряд. Между заряженным стержнем и цинковым сосудом возникает
электрическое поле. В таком источнике тока уголь является положительным
электродом, а цинковый сосуд — отрицательным электродом.
Из нескольких гальванических элементов можно составить батарею.
Источники тока на основе гальванических элементов применяются в
бытовых автономных электроприборах, источниках бесперебойного
питания.
Аккумуляторы — в автомобилях, электромобилях, сотовых телефонах.

19. Закон Ома для полной цепи

Сила тока (А)
I
Сопротивление
нагрузки (Ом)
R r
Сила тока в цепи прямо
пропорциональна электродвижущей силе
источника тока и обратно
пропорциональна сумме электрических
сопротивлений внешнего и внутреннего
участков цепи.
ЭДСэлектродвижущая
сила источника тока
(В)
Внутреннее
сопротивление
источника тока
(Ом)

20. Проведите аналогию

I
R r
U
I
R

21. Если на участке цепи не действует ЭДС (нет источника тока)

U=φ1-φ2
Если концы участка, содержащего источник тока, соединить,
то их потенциал станет одинаков
U=ε
В замкнутой цепи напряжение на внешнем и внутреннем ее
участках равно ЭДС источника тока
ε=Uвнеш+Uвнутр

22. Короткое замыкание

При коротком замыкании R → 0,
сила тока
I
R r
I кз
r

23. Виды предохранителей


Плавкие
Автоматические
Сетевые фильтры
Щитки автоматические
Щиток автоматический

24. Вычислите токи короткого замыкания

Источник тока
Гальванический
элемент
Аккумулятор
Осветительные
сети
ε,В
r, Ом
Iк.з., А
1,5
1
1,5
6
0,01
600
100
0,001
100 000
1.
Вычислите силу тока в спирали электрической
плитки, включенной в сеть с напряжением 220В,
если сопротивление спирали равно 100 Ом.
2. Сила тока , проходящая через нить лампы 0,3 А,
напряжение лампы 6 В. Какое электрическое
сопротивление нити лампы?
3. Сила тока в цепи 2 А, сопротивление резистора 110
Ом. Чему равно напряжение в цепи?

26. Решение задач:

№1 Гальванический элемент с ЭДС E = 5,0 В и
внутренним сопротивлением r = 0,2 Ом замкнут на
проводник сопротивлением R = 40,0 Ом. Чему равно
напряжение U на этом проводнике?
№2 К аккумулятору с ЭДС 12 В
и внутренним сопротивлением r =0,5 Ом,
подключили лампочку сопротивлением R=100 Ом.
Определить силу тока в цепи.
№3 Определить ЭДС источника тока с внутренним
сопротивлением r = 0,3 Ом, если при подключении
к клеммам источника тока параллельно соединенных
резисторов R1=10 Ом и R2=6 Ом сила тока в цепи:
I=3 A.

27. Решение задач:

№1 Гальванический элемент с ЭДС E = 5,0 В и
внутренним сопротивлением r = 0,2 Ом замкнут на
проводник сопротивлением R = 40,0 Ом. Чему равно
напряжение U на этом проводнике?
Ответ: U = 4,97 В.
№2 К аккумулятору с ЭДС 12 В
и внутренним сопротивлением r =0,5 Ом,
подключили лампочку сопротивлением R=100 Ом.
Определить силу тока в цепи.
Ответ: 0,119 А
№3 Определить ЭДС источника тока с внутренним
сопротивлением r = 0,3 Ом, если при подключении
к клеммам источника тока параллельно соединенных
резисторов R1=10 Ом и R2=6 Ом сила тока в цепи:
I=3 A.
Ответ: 12,15В

28. Тест

• 1 Формула выражающая закон Ома для замкнутой
цепи записывается как:
а) I=U/R
б) I
в)
I
R r
R r 2
г) I
R r

29. Тест

2.Ток короткого
формуле:
U
Ik
R
а)
б)
Ik r
в)
Ik
г)
U
Ik
r
r
замыкания
можно
рассчитать
по

30. Тест (готовимся к ЕГЭ)

3.ЭДС аккумулятора с внутренним
сопротивлением r =0,2 Ом, при
подключении к нему сопротивления
R=5 Ом равно…
По цепи протекает ток I=1,5 A.
А) 3 В
Б) 12В
В) 7,8 В
Г) 12,2В

31. Тест (готовимся к ЕГЭ)

4.Какое внутреннее сопротивление имеет источник
тока с ЭДС 12 В, если при замыкании его
параллельно соединенными резисторами R 13
1
Ом и R 7 Ом в цепи протекает ток I=2 A.
2
А) 26 Ом
Б) 1,45 Ом
В) 12 Ом
Г) 2,45 Ом

32. Ответы на тест:

• №1
• Г
№2
В
№3
В
№4
Б

33. Рефлексия

А. Мне все понравилось. Я все понял
Б. Мне понравилось, но я не все понял
В. Все как всегда, ничего необычного
Г. Мне не понравилось

34. Домашнее задание

§ 107-108 читать,упр 19 №5,6.
Задача (на дом):
При подключении лампочки к батарее
элементов с ЭДС 4,5 В вольтметр
показал напряжение на лампочке 4 В, а
амперметр – силу тока 0,25 А. Каково
внутреннее сопротивление батареи?
Спасибо за урок!

Закон Ома для полной цепи | ESP32 Arduino

Чтобы понять, что такое закон Ома для полной электрической цепи, соберем простую схему состоящую из трех элементов:

У нас есть батарея ЭДС которой составляет E = 8.75 В

ЭДС батареи, можно измерить вольтметром без подключения нагрузки (в школьных учебниках обычно обозначается буквой ε э́псилон)

ЭДС батареи, можно измерить вольтметром без подключения нагрузки (в школьных учебниках обычно обозначается буквой ε э́псилон)

Сопротивление номиналом R = 100 Ом

Сопротивление номиналом 100 Ом

Сопротивление номиналом 100 Ом

Какой ток покажет амперметр в данной схеме?

Схема для проверки закона ома для полной цепи

Схема для проверки закона ома для полной цепи

Исходя из формулировки закона Ома: Сила тока в участке цепи прямо пропорциональна напряжению и обратно пропорциональна электрическому сопротивлению данного участка цепи, можно попробовать посчитать ток в цепи по знакомой всем формуле: I=U/R

расчет по формуле закона Ома для участка цепи

расчет по формуле закона Ома для участка цепи

Но, в реальной жизни и с реальными источниками питания эта простая формула не сработает. После подключения резистора напряжение на клеммах батареи просело до значения 8.07 В, а ток составляет всего лишь 80 мА (0.080A)

Изучение закона Ома для полной цепи

Изучение закона Ома для полной цепи

Вывод здесь достаточно простой. Раз ток в цепи не соответствует нашим ожиданиям, после подключения нагрузки, в цепи где то появилось дополнительное сопротивление (провода откидываем сразу — они медные толстые и короткие). Очевидно, что это сопротивление находится внутри самой батареи (больше негде), а значит осталось нарисовать реальную схему, и вычислить это внутреннее сопротивление.

Алгоритм расчета внутреннего сопротивления батареи

Алгоритм расчета внутреннего сопротивления батареи

Формулировка закона Ома для полной цепи звучит следующим образом — «Сила тока в полной цепи равна отношению ЭДС цепи к ее полному сопротивлению». Мы уже опытным путем выяснили, что в нашей цепи у нас два сопротивления (одно внутри батареи, а второе, то которое подключили к клеммам батареи). Отсюда, формула закона Ома для полной цепи вырисовывается как бы сама собой.

Закон ома для полного участка цепи формула

Закон ома для полного участка цепи формула

R – внешнее сопротивление [Ом];
r – сопротивление источника ЭДС (внутреннее) [Ом];
I – сила тока [А];
E– ЭДС источника тока [В].

Мы уже путем измерений и вычислений выяснили внутреннее сопротивление нашего источника питания r = 8.5 Ом.

Проверка закон ома для полного участка цепи

Проверка закон ома для полного участка цепи

Для нашего практического случая это и есть ответ на вопрос заданный в начале этой статьи и именно этот ток и показал нам амперметр.

Любой реальный источник тока обладает внутренним сопротивлением, которое обусловлено либо сопротивлением растворов электролитов для гальванических элементов и аккумуляторов, либо сопротивлением проводников для генераторов.

Именно внутреннее сопротивление ограничивает ток в цепи при коротком замыкании!

Для обычных батареек такие понятия как села батарея (разрядилась) в физическом смысле означают, что у батареи увеличилось внутреннее сопротивление. Выяснить это проще всего измерив ток короткого замыкания.

Измерение производить кратковременно! Категорически запрещается измерять ток короткого замыкания у аккумуляторов и сетевых блоков питания — многие из этих источников питания изначально созданы для больших токовых нагрузок. В лучшем случае сгорит амперметр или источник питания выйдет из строя — в худшем будет пожар или взрыв.

К примеру ЭДС этой довольно свежей 9 В батареи E = 8.57 В

Напряжение на батарее без нагрузки

Напряжение на батарее без нагрузки

Ток короткого замыкания в пике 1.33A

Ток короткого замыкания, измеряем кратковременно чтобы не разрядить батарею!

Ток короткого замыкания, измеряем кратковременно чтобы не разрядить батарею!

Внутреннее сопротивление батареи:

Внутреннее сопротивление свежей батареи.

Внутреннее сопротивление свежей батареи.

Эта батарея еще поработает….

ЭДС этой батареи 7.27V

Напряжение на батарее без нагрузки

Напряжение на батарее без нагрузки

Ток короткого замыкания в пике всего 0.5A

Ток короткого замыкания, измеряем кратковременно чтобы не разрядить батарею!

Ток короткого замыкания, измеряем кратковременно чтобы не разрядить батарею!

Внутреннее сопротивление севшей (разряженной) батареи.

Внутреннее сопротивление севшей (разряженной) батареи.

Подключив от такой батареи устройство с электроникой требовательной к уровню напряжению питания, посадка которая возникнет в результате внутреннего сопротивления батареи может привести к тому, что устройство будет работать некорректно.

Разумеется то же самое касается и подбора источников питания для самодельных электронных устройств. Мало подобрать источник питания исходя из его ЭДС — необходимо чтобы он еще обладал таким внутренним сопротивлением, чтобы мог «тянуть» подключенную к нему нагрузку.

Полный список статей канала доступен по этой ссылке

17.4. Закон Ома в интегральной форме

Для любой точки внутри проводника напряженность результирующего поля равна сумме напряженности поля кулоновских сил и поля сторонних сил . Подставляя в (17.6), получим

Умножим скалярно обе части на вектор , численно равный элементу длины проводника и направленный по касательной к проводнику в ту же сторону, что и вектор плотности тока

Так как скалярное произведение совпадающих по направлению векторов и , равно произведению их модулей, то это равенство можно переписать в виде


С учетом

Интегрируя по длине проводника от сечения 1 до некоторого сечения 2 и учитывая, что сила тока во всех сечениях проводника одинакова, получаем

(17.7)

Интеграл численно равен работе, совершаемой кулоновскими силами при перенесении единичного положительного заряда с точки 1 в точку 2. В электростатике было показано, что


Таким образом,

где и — значение потенциала в т.1 и т.2.

Интеграл, содержащий вектор напряженности поля, сторонних сил, представляет собой эдс , действующей на участке 1-2

(17.9)

Интеграл

(17.10)

равен сопротивлению участка цепи 1-2.

Подставляя (17.10), (17.9) и (17.8) в (17.7), окончательно получим

(17.11)

Последнее уравнение выражает собой закон Ома в интегральной форме для участка цепи, содержащего эдс и формулируется следующим образом: падение напряжения на участке цепи равно сумме падений электрического потенциала на этом участке и эдс всех источников электрической энергии, включённых на участке.

При замкнутой внешней цепи сумма падений электрических потенциалов и эдс источника равна сумме падений напряжения на внутреннем сопротивлении источника и во всей внешней цепи где или Отсюда

(17.12)

Электродвижущая сила. Закон Ома для полной цепи

Как вы знаете, для существования электрического тока, необходимо наличие электрического поля. Причем, это поле должно постоянно поддерживаться неким источником тока. Сегодня мы поговорим об основной характеристике источника тока, которая называется электродвижущей силой (или, сокращенно, ЭДС). Для начала рассмотрим простой опыт: возьмем два противоположно заряженных шарика и соединим их проводником. В этом случае, в проводнике возникнет электрический ток, но он будет очень кратковременным. Дело в том, что очень скоро произойдет перераспределение заряда, и потенциалы шариков уравняются. Значит, перестанет существовать электрическое поле.

Из этого можно сделать вывод, что для поддержания постоянного тока необходимо наличие неких сил неэлектрического происхождения, чтобы эти силы могли перемещать заряды против поля. Такие силы называются сторонними силами. То есть, сторонние силы — это любые силы, которые действуют на электрические заряды, но при этом не являются силами электрического происхождения. Например, это могут быть силы, действующие на заряды со стороны магнитного поля — это используется в генераторах.

В батареях или аккумуляторах работу по разделению электрических зарядов выполняют химические реакции.

Еще один аргумент, который мы можем привести — это то, что работа кулоновских сил при перемещении заряда по замкнутому контуру, равна нулю. А это значит, что какие-то другие силы должны обеспечивать ненулевую работу для поддержания разности потенциалов.

Устройство для поддержания электрического тока, называется источником тока. В любом источнике тока сторонние силы действуют на заряды, совершая работу против кулоновских сил. Стало быть, характеристикой источника должна быть величина, не зависящая от величины заряда. Эта величина называется электродвижущей силой. Электродвижущая сила равна отношению работы сторонних сил при перемещении заряда по замкнутому контуру, к величине этого заряда:

Из формулы видно, что электродвижущая сила, как и напряжение, измеряется в вольтах:

Теперь, когда мы познакомились с ЭДС, мы можем перейти к изучению закона Ома для полной цепи. Полной цепью называется замкнутая цепь, включающая в себя источник тока. Для удобства, мы рассмотрим простейшую электрическую цепь, состоящую только из источника тока, резистора и соединительных проводов:

Как мы уже сказали, источник тока характеризуется ЭДС. Тем не менее, любой источник тока обладает определенным сопротивлением, которое называется внутренним сопротивлением. Закон Ома для полной цепи представляет собой связь между ЭДС, внутренним и внешним сопротивлением и силой тока в цепи. Для того, чтобы установить эту связь, воспользуемся законом сохранения энергии. Запишем, что работа сторонних сил равна произведению ЭДС источника и величины заряда:

Как вы знаете, каждый участок цепи выделяет то или иное количество теплоты. По закону Джоуля-Ленца, это количество теплоты вычисляется по формуле:

Исходя из закона сохранения энергии, мы можем приравнять это количество теплоты к работе сторонних сил:

Закон Ома для полной цепи звучит так: сила тока в замкнутой цепи равна отношению ЭДС источника к полному сопротивлению цепи:

Вывести закон Ома для полной цепи можно, рассуждая несколько иначе. Как мы знаем, при последовательном соединении полное напряжение цепи равно сумме падений напряжений на всех участках цепи:

Мы видим, что произведение силы тока и сопротивления резистора есть не что иное, как напряжение на этом резисторе. А произведение силы тока и внутреннего сопротивления — это падение напряжения на самом источнике:

Надо сказать, что внутреннее сопротивление источника во многих случаях пренебрежимо мало по сравнению с сопротивлением внешней части цепи. В этом случае, мы можем считать, что напряжение на зажимах источника примерно равно ЭДС (то есть падение напряжения на источнике считается приблизительно равным нулю):

Тем не менее, именно внутренним сопротивлением определяется сила тока в цепи при коротком замыкании. Напомним, что при коротком замыкании, внешнее сопротивление становится почти нулевым, поэтому в цепи резко возрастает сила тока:

Рассмотрим теперь цепь, содержащую несколько последовательно соединенных источников тока.

В этом случае, ЭДС всей цепи равна алгебраической сумме ЭДС отдельных источников.

В таких случаях необходимо выбрать так называемое «направление обхода тока». Это направление выбирается условно (в нашем случае — против часовой стрелки). Тогда, ,поскольку они стремятся вызвать ток в направлении обхода.

А,поскольку они стремятся вызвать ток в направлении, противоположном направлению обхода. Отрицательная ЭДС означает, что сторонние силы внутри источника совершают отрицательную работу. Таким образом, ЭДС нашей цепи будет равна:

В соответствии с правилами последовательного соединения, суммарное сопротивление цепи равно сумме внешнего сопротивления и внутренних сопротивлений всех источников тока:

Пример решения задачи.

Задача. К источнику тока с внутренним сопротивлением 1 Ом подключили резистор с сопротивлением 15 Ом. После этого в цепь включили амперметр, который показал, что сила тока равна 5 А. Найдите работу сторонних сил внутри источника, совершенную за 2 минуты.

Объясните применение закона Ома к полному кругу физики класса 10 CBSE

Подсказка: Ом дали соотношение между тремя величинами: током, напряжением и сопротивлением цепи. Закон Ома гласит, что напряжение или разность потенциалов между двумя точками прямо пропорциональна току в цепи. Нарисуйте линейную цепь, содержащую батарею и сопротивление, и определите ток в цепи.

Полный шаг за шагом Ответ:
У нас есть закон Ома, который гласит, что напряжение или разность потенциалов между двумя точками прямо пропорциональны току в цепи.Следовательно, мы можем выразить закон Ома как:
\ [V \ propto I \]
\ [\ Rightarrow V = IR \]
Где V — напряжение, I — ток, проходящий через цепь. Константа пропорциональности — это сопротивление цепи R.

Немецкий ученый Джордж Саймон Ом обнаружил взаимосвязь между током, напряжением и сопротивлением в цепи. Используя закон Ома, мы можем определить ток, протекающий по цепи, если сопротивление в цепи и ЭДС источника питания известны. нас.Например, давайте рассмотрим схему ниже.


В приведенной выше схеме сопротивление R подключено последовательно с батареей постоянного тока с ЭДС 12 В. Используя закон Ома, мы можем вычислить ток в указанной выше схеме как:
\ [V = IR \]
\ [\ Стрелка вправо 12 = \ left (I \ right) \ left ({200} \ right) \]
\ [\ поэтому I = 0,06 \, {\ text {A}} \]
Таким образом, неизвестный ток равен 0,06 ампера.

Дополнительная информация:
Мы знаем, что согласно закону Ома напряжение / ток = $ a $ константа, и эта константа — это сопротивление.Но сопротивление не остается постоянным. При изменении температуры материала изменяется и сопротивление. Закон не распространяется на односторонние сети. Односторонние сети позволяют току течь в одном направлении. Закон Ома также не применим к нелинейным элементам. Соотношение между V и I зависит от знака V, где V — напряжение, а I — ток в цепи.

Примечание. Закон Ома не применяется к односторонним сетям, в которых используются диоды, передатчики и т. Д.Если у нас есть несколько резисторов в цепи, закон Ома все еще применим. Чтобы определить ток, первое, что нам нужно сделать, это определить полное сопротивление в цепи. Если в цепи две ЭДС, мы не можем следовать закону Ома.

Закон Ома

— StatPearls — Книжная полка NCBI

Введение

Закон Ома — это взаимосвязь между тремя физическими явлениями: током, напряжением и сопротивлением. Ток определяется как поток положительного заряда от источника к источнику отрицательного заряда.Единицами измерения тока являются Кл / с для количества заряда (Кл), который проходит за единицу времени (с). Ампер (А) — это обычная единица измерения тока, равная 1 Кл / с, а символом тока является I. Ток является внутренним свойством, так как он зависит от других аспектов, таких как размер системы. Чтобы точно сравнить величину тока для разных систем, ток нормализован по площади или массе системы. Это описывается следующим образом:

Где J — плотность тока в л / (м · м) или л / г, в зависимости от того, как сравниваются системы, I — ток (A), A — площадь поперечного сечения. (m · m), а m — масса (г).Обратите внимание, что часто j используется для тока вместо I, чтобы избежать путаницы с мнимыми числами. Поэтому следует обратить внимание на определения символов, так как они могут варьироваться в зависимости от случая.

Напряжение — это еще одна часть закона Ома, который устанавливает объем работы, необходимый для перемещения заряда. Единица измерения напряжения — Дж / Кл, что равно широко распространенной единице Вольт (В). Напряжение измеряет электрический потенциал объекта по отношению к заряду. Путем подачи напряжения на заряд совершается работа, которая обеспечивает движение заряда.Количество заряда по сравнению с индивидуальным зарядом, известное как точечный заряд, может быть определено следующим образом:

Где V — электрический потенциал (В), k — постоянная 8,99 E 9 Н · м · м / (C · C), q — заряд точки (C), а r — расстояние от точечного заряда (м).

Сопротивление — это противостояние движению заряда. Сопротивление аналогично эффектам трения в текущей воде или скользящем предмете. Единицы измерения сопротивления — Ом, что обозначается заглавной греческой буквой Омега.Чтобы вычислить величину сопротивления в объекте, можно использовать следующее уравнение:

Где R — сопротивление (Омега), Rho — удельное сопротивление объекта (Омега · м), l — длина объекта (м ), а A — площадь поперечного сечения объекта (м · м). Удельное сопротивление различно для каждого объекта и зависит от структуры материала. Расчет удельного сопротивления выходит за рамки данной статьи.

Сопротивление также можно нормализовать, чтобы обеспечить точное сравнение в каждом конкретном случае.Нормализованное сопротивление определяется по формуле:

, где R — нормализованное сопротивление (Омега · м · м). Сопротивление, препятствующее прохождению заряда, обратно пропорционально току. Поскольку текущая нормализация относится к единицам площади, нормализация сопротивления умножается на единицы площади поперечного сечения из-за обратной зависимости.

Сопротивление, обратное сопротивлению (1 / R), известно как проводимость, которая измеряет способность объекта проводить заряд, выраженный в единицах Сименс (S).Дальнейшее обсуждение поведения выходит за рамки данной статьи; Однако стоит отметить обратную зависимость проводимости от сопротивления.

Учитывая ток, напряжение и сопротивление, закон Ома определяется как:

Размерный анализ необходим для обеспечения согласованности единиц. [1] [2] [3] [4]

Функция

Закон Ома изначально был эмпирическим выводом, основанным на соотношении между током и напряжением; тем не менее, он оставался неизменным в микроскопических и макроскопических наблюдениях.Закон Ома широко используется в электронике и при создании схем, а также при разработке аккумуляторов и других электрохимических приложениях. Сфера применения закона Ома не ограничивается схемами, так как его также можно использовать для создания молекул и полимеров. Каждый атом содержит протоны и электроны, которые можно использовать для создания химических связей с другими атомами. При приложении напряжения к материалу происходит окислительно-восстановительная реакция, когда один электрон удаляется из одной молекулы и присоединяется к другой.У этого есть множество исследовательских приложений, таких как хранение энергии, химия материалов, органическая химия и многое другое. Эти приложения используют закон Ома и позволили ученым создавать материалы новыми способами.

Чтобы выбрать правильное напряжение, необходимо учитывать сопротивление, а также количество необходимого заряда. Если сопротивление велико, а напряжение мало, тока не хватит для подачи заряда повсюду. И наоборот, если напряжение слишком велико, ток может повредить схему устройства или разрушить молекулы.Использование электричества по закону Ома требует внимания к потребностям приложения для получения желаемой мощности.

Проблемы, вызывающие озабоченность

Основная проблема, вызывающая озабоченность в связи с законом Ома, заключается в том, что расчеты могут усложняться из-за сложных схем и электрохимических явлений. Например, общее сопротивление состоит из различных типов сопротивления, таких как омическое сопротивление, сопротивление массопереносу и сопротивление переносу заряда. При оптимизации электрических характеристик устройств существует компромисс: уменьшение одного сопротивления может увеличить другое.Это главное соображение в исследованиях проектирования и оптимизации, чтобы найти, какие значения лучше всего подходят для данного приложения. Часто эти различные типы сопротивления трудно измерить, и для получения точных значений требуются сложные машины. Кроме того, сопротивление также изменяется со временем, поскольку структурная целостность устройства ухудшается. Это также может изменить ожидаемые результаты закона Ома, если это не учитывать. [5] [6]

Клиническая значимость

Закон

Ом используется в каждом электрическом устройстве для приложения правильного количества заряда с учетом сопротивления.Хотя клиницистам не нужно быстро рассчитывать напряжение, ток и сопротивление для этих устройств, понимание взаимосвязи может дать интуитивное представление о том, как работают электрические устройства. Закон Ома также присутствует в человеческом теле и имеет множество применений. Например, нервы нервной системы используют электрические свойства для передачи сообщений друг другу. Эти электрические сигналы могут быть отображены датчиками и лучше поняты клиницистами. Gao et al. и Su et al. изучили электрические свойства костей и продвинулись в лечении заболеваний костей.В человеческом теле есть множество электрических путей и процессов. Понимая фундаментальное управляющее уравнение, врачи могут работать над решением проблем по-новому.

Улучшение результатов команды здравоохранения

Закон Ома — это взаимосвязь между тремя физическими явлениями: током, напряжением и сопротивлением. Ток определяется как поток положительного заряда от источника к источнику отрицательного заряда. Закон Ома изначально был эмпирическим выводом, основанным на соотношении между током и напряжением; тем не менее, он оставался неизменным в микроскопических и макроскопических наблюдениях.Закон Ома широко используется в электронике и при создании схем, а также при разработке аккумуляторов и других электрохимических приложениях.

19,1 Закон Ома | Texas Gateway

Постоянный и переменный ток

Так же, как вода течет с большой высоты на низкую, электроны, которые могут свободно перемещаться, будут перемещаться из места с низким потенциалом в место с высоким потенциалом. Батарея имеет две клеммы с разным потенциалом. Если клеммы соединены проводом, электрический ток (заряды) будет течь, как показано на рисунке 19.2. Электроны будут перемещаться от низкопотенциальной клеммы батареи (отрицательный конец ) по проводу и попадут в высокопотенциальную клемму батареи (положительный конец ).

Рисунок 19.2 У батареи есть провод, соединяющий положительную и отрицательную клеммы, который позволяет электронам перемещаться от отрицательной клеммы к положительной.

Электрический ток — это скорость движения электрического заряда. Большой ток, такой как тот, который используется для запуска двигателя грузовика, перемещает большое количество очень быстро, тогда как небольшой ток, такой как тот, который используется для работы портативного калькулятора, перемещает небольшое количество заряда медленнее.В форме уравнения электрический ток I определяется как

, где ΔQΔQ — это количество заряда, которое проходит через заданную область, а ΔtΔt — время, за которое заряд проходит мимо этой области. Единицей измерения электрического тока в системе СИ является ампер (А), названный в честь французского физика Андре-Мари Ампера (1775–1836). Один ампер — это один кулон в секунду, или

1 А = 1 Кл / с. 1 А = 1 Кл / с.

Электрический ток, движущийся по проволоке, во многом похож на ток воды, движущийся по трубе.Чтобы определить поток воды через трубу, мы можем подсчитать количество молекул воды, которые проходят мимо данного участка трубы. Как показано на рисунке 19.3, электрический ток очень похож. Мы подсчитываем количество электрических зарядов, протекающих по участку проводника; в данном случае провод.

Рис. 19.3 Электрический ток, движущийся по этому проводу, — это заряд, который проходит через поперечное сечение A, деленный на время, необходимое этому заряду, чтобы пройти через участок A .

Предположим, что каждая частица q на рисунке 19.3 несет заряд q = 1 nCq = 1 нКл, и в этом случае показанный общий заряд будет ΔQ = 5q = 5 nCΔQ = 5q = 5 нКл. Если эти заряды пройдут область A и за время Δt = 1 нсΔt = 1 нс, то ток будет

19.1I = ΔQΔt = 5 nC1 ns = 5 A.I = ΔQΔt = 5 nC1 ns = 5 A.

Обратите внимание, что мы присвоили зарядам на рис. 19.3 положительный заряд. Обычно отрицательные заряды — электроны — являются подвижным зарядом в проводах, как показано на рисунке 19.2. Положительные заряды обычно застревают в твердых телах и не могут свободно перемещаться.Однако, поскольку положительный ток, движущийся вправо, аналогичен отрицательному току такой же величины, движущемуся влево, как показано на рисунке 19.4, мы определяем обычный ток, который течет в том направлении, в котором протекал бы положительный заряд, если бы он мог двигаться. . Таким образом, если не указано иное, предполагается, что электрический ток состоит из положительных зарядов.

Также обратите внимание, что один кулон — это значительная величина электрического заряда, поэтому 5 А — это очень большой ток. Чаще всего вы увидите ток порядка миллиампер (мА).

Рис. 19.4 (a) Электрическое поле направлено вправо, ток движется вправо, а положительные заряды движутся вправо. (б) Эквивалентная ситуация, но с отрицательными зарядами, движущимися влево. Электрическое поле и ток по-прежнему справа.

Snap Lab

Vegetable Current

Эта лабораторная работа помогает студентам понять, как работает ток. Учитывая, что частицы, заключенные в трубе, не могут занимать одно и то же пространство, толкание большего количества частиц в один конец трубы приведет к вытеснению того же количества частиц из противоположного конца.Это создает поток частиц.

Найдите солому и сушеный горох, которые могут свободно перемещаться в соломе. Положите соломинку на стол и засыпьте ее горошком. Когда вы вдавливаете одну горошину с одного конца, другая горошина должна выходить из другого конца. Эта демонстрация представляет собой модель электрического тока. Определите часть модели, которая представляет электроны, и часть модели, которая представляет собой подачу электроэнергии. В течение 30 секунд подсчитайте, сколько горошин вы можете протолкнуть через соломинку.Когда закончите, рассчитайте горошин, текущий , разделив количество горошин на время в секундах.

Обратите внимание, что поток гороха основан на том, что горох физически сталкивается друг с другом; электроны толкают друг друга за счет взаимно отталкивающих электростатических сил.

Проверка захвата

Предположим, четыре горошины в секунду проходят через соломинку. Если бы каждая горошина несла заряд 1 нКл, какой электрический ток проходил бы через соломинку?

  1. Электрический ток будет равен заряду гороха, умноженному на 1 нКл / горох.
  2. Электрический ток будет равняться пиковому току, вычисленному в лаборатории, умноженному на 1 нКл / горох.
  3. Электрический ток будет равняться гороховому току, рассчитанному в лаборатории.
  4. Электрический ток равен заряду горошины, разделенному на время.

Направление обычного тока — это направление, в котором будет течь положительный заряд . В зависимости от ситуации могут перемещаться положительные заряды, отрицательные заряды или и то, и другое.В металлических проводах, как мы видели, ток переносится электронами, поэтому отрицательные заряды движутся. В ионных растворах, таких как соленая вода, движутся как положительно заряженные, так и отрицательно заряженные ионы. То же самое и с нервными клетками. Чистые положительные токи относительно редки, но встречаются. История отмечает, что американский политик и ученый Бенджамин Франклин описал ток как направление, в котором положительные заряды проходят через провод. Он назвал тип заряда, связанный с электронами, отрицательным задолго до того, как стало известно, что они переносят ток во многих ситуациях.

Когда электроны движутся по металлической проволоке, они сталкиваются с препятствиями, такими как другие электроны, атомы, примеси и т. Д. Электроны рассеиваются от этих препятствий, как показано на рисунке 19.5. Обычно электроны теряют энергию при каждом взаимодействии. Таким образом, чтобы электроны двигались, требуется сила, создаваемая электрическим полем. Электрическое поле в проводе направлено от конца провода с более высоким потенциалом к ​​концу провода с более низким потенциалом. Электроны, несущие отрицательный заряд, движутся в среднем ( дрейф ) в направлении, противоположном электрическому полю, как показано на рисунке 19.5.

Рис. 19.5. Свободные электроны, движущиеся в проводнике, совершают множество столкновений с другими электронами и атомами. Показан путь одного электрона. Средняя скорость свободных электронов находится в направлении, противоположном электрическому полю. Столкновения обычно передают энергию проводнику, поэтому для поддержания постоянного тока требуется постоянный запас энергии.

До сих пор мы обсуждали ток, который постоянно движется в одном направлении. Это называется постоянным током, потому что электрический заряд течет только в одном направлении.Постоянный ток часто называют постоянным током током.

Многие источники электроэнергии, такие как плотина гидроэлектростанции, показанная в начале этой главы, вырабатывают переменный ток, направление которого меняется взад и вперед. Переменный ток часто называют Переменный ток . Переменный ток перемещается вперед и назад через равные промежутки времени, как показано на рисунке 19.6. Переменный ток, который исходит из обычной розетки, не меняет направление внезапно.Скорее, он плавно увеличивается до максимального тока, а затем плавно уменьшается до нуля. Затем он снова растет, но в противоположном направлении, пока не достигнет того же максимального значения. После этого он плавно уменьшается до нуля, и цикл начинается снова.

Рисунок 19.6 При переменном токе направление тока меняется через определенные промежутки времени. График вверху показывает зависимость тока от времени. Отрицательные максимумы соответствуют движению тока влево.Положительные максимумы соответствуют току, движущемуся вправо. Ток регулярно и плавно чередуется между этими двумя максимумами.

Устройства, использующие переменный ток, включают пылесосы, вентиляторы, электроинструменты, фены и многие другие. Эти устройства получают необходимую мощность, когда вы подключаете их к розетке. Настенная розетка подключена к электросети, которая обеспечивает переменный потенциал (потенциал переменного тока). Когда ваше устройство подключено к сети, потенциал переменного тока толкает заряды вперед и назад в цепи устройства, создавая переменный ток.

Однако во многих устройствах используется постоянный ток, например в компьютерах, сотовых телефонах, фонариках и автомобилях. Одним из источников постоянного тока является аккумулятор, который обеспечивает постоянный потенциал (потенциал постоянного тока) между своими выводами. Когда ваше устройство подключено к батарее, потенциал постоянного тока толкает заряд в одном направлении через цепь вашего устройства, создавая постоянный ток. Другой способ получения постоянного тока — использование трансформатора, который преобразует переменный потенциал в постоянный. Небольшие трансформаторы, которые вы можете подключить к розетке, используются для зарядки вашего ноутбука, мобильного телефона или другого электронного устройства.Люди обычно называют это зарядным устройством или батареей , но это трансформатор, который преобразует напряжение переменного тока в напряжение постоянного тока. В следующий раз, когда кто-то попросит одолжить зарядное устройство для ноутбука, скажите им, что у вас нет зарядного устройства для ноутбука, но они могут одолжить ваш преобразователь.

Рабочий пример

Ток при ударе молнии

Удар молнии может передать до 10201020 электронов из облака на землю. Если удар длится 2 мс, каков средний электрический ток в молнии?

СТРАТЕГИЯ

Используйте определение тока, I = ΔQΔtI = ΔQΔt.Заряд ΔQΔQ из 10201020 электронов ΔQ = neΔQ = ne, где n = 1020n = 1020 — количество электронов, а e = −1.60 × 10−19 Ce = −1.60 × 10−19 C — заряд электрона. Это дает

19,2 ΔQ = 1020 × (−1,60 × 10−19 ° C) = — 16,0 ° C ΔQ = 1020 × (−1,60 × 10−19 ° C) = — 16,0 ° C.

Время Δt = 2 × 10–3 с Δt = 2 × 10–3 с — это продолжительность удара молнии.

Решение

Ток при ударе молнии

19,3I = ΔQΔt = −16,0 C2 × 10−3 с = −8 кА. I = ΔQΔt = −16,0 C2 × 10−3 с = −8 кА.

Обсуждение

Отрицательный знак отражает тот факт, что электроны несут отрицательный заряд.Таким образом, хотя электроны текут от облака к земле, положительный ток должен течь от земли к облаку.

Рабочий пример

Средний ток для заряда конденсатора

В цепи, содержащей конденсатор и резистор, заряд конденсатора емкостью 16 мкФ с использованием батареи 9 В. занимает 1 мин. Какой средний ток в это время?

СТРАТЕГИЯ

Мы можем определить заряд конденсатора, используя определение емкости: C = QVC = QV.Когда конденсатор заряжается батареей 9 В, напряжение на конденсаторе будет V = 9 VV = 9 В. Это дает заряд

.

Подставляя это выражение для заряда в уравнение для тока, I = ΔQΔtI = ΔQΔt, мы можем найти средний ток.

Решение

Средний ток

19,5I = ΔQΔt = CVΔt = (16 × 10−6 F) (9 В) 60 с = 2,4 × 10−6 A = 2,4 мкА I = ΔQΔt = CVΔt = (16 × 10−6 F) (9 В) 60 с = 2,4 × 10-6 А = 2,4 мкА.

Обсуждение

Этот небольшой ток типичен для тока, встречающегося в таких цепях.

Что такое закон Ома? Объяснение и ограничения закона Ома

Когда разность электрических потенциалов (В) приложена к проводнику, как показано на рисунке ниже, через него протекает некоторый ток (I) . Протеканию тока препятствует сопротивление проводника и цепи. Связь между напряжением, током и сопротивлением объясняется законом Ома.

Законы

Ома гласят, что ток через любые две точки проводника прямо пропорционален разности потенциалов, приложенной к проводнику, при условии физических условий: i.е. температура и т. д. не меняются. Оно измеряется в ( Ом, ) Ом.

Математически это выражается как

Другими словами, закон Ома также можно сформулировать как;

Отношение разности потенциалов в конечной точке проводника к току, протекающему между ними, всегда постоянно, но физические условия проводника, то есть температура и т. Д., Остаются неизменными.

Эта постоянная также называется сопротивлением (R) проводника (или цепи)

Его можно записать как

В цепи, когда ток течет через резистор, разность потенциалов на резисторе известна как падение напряжения на нем, т.е.е., В = ИК.

Ограничения закона Ома

  • Закон Ома не применяется в односторонних сетях. Односторонние сети позволяют току течь в одном направлении. Такие типы сетей состоят из таких элементов, как диод, транзистор и т. Д.
  • Это не применимо для нелинейной сети. В нелинейной сети параметр сети изменяется в зависимости от напряжения и тока. Их параметр включает сопротивление, индуктивность, емкость, частоту и т. Д., не оставаться неизменным со временем. Так что закон Ома неприменим к нелинейной сети.

Закон Ома используется для определения сопротивления цепи, а также для определения напряжения и тока цепи.

Закон ужасного ома

В предыдущей статье мы рассмотрели формулу мощности и обсудили взаимосвязь между мощностью, напряжением и током. Посмотрев на формулу мощности, теперь есть только одна формула, необходимая для решения практически всех электрических расчетов, это ужасный закон Ома.Мы не будем вдаваться в вывод формулы, но уделите минуту, чтобы прочитать следующую иллюстрацию:

Когда вы откроете кран, подключенный к садовому шлангу без насадки, вы увидите, что вода вытекает из другого конца. Поток воды через шланг подобен потоку электрического тока — электрический ток — это поток электронов через провод.

Если теперь вы положите большой палец на конец шланга и попытаетесь перекрыть поток воды, давление воды, которое вы почувствуете, будет похоже на напряжение.Напряжение — это давление, которое толкает электроны по проводу. Что интересно, если ваш большой палец полностью перекрывает поток воды, давление (напряжение) воды все равно остается. Наличие насадки на конце шланга и прекращение потока воды похоже на то, что к розетке в стене ничего не подключено — нет воды / тока, но давление / напряжение все еще есть. Следует отметить, что напряжение присутствует, даже если нет тока. Другой пример — автомобильный аккумулятор — на любой подключенный к нему провод всегда подается 12 Вольт, независимо от того, какой ток идет.

Вернемся к садовому шлангу. Если теперь переместить большой палец на конец шланга, можно изменить количество выходящей воды. То есть, изменяя сопротивление воде, вы меняете и поток. То же самое и в электрической цепи — измените сопротивление и соответственно изменится ток. Давайте определим электрическое сопротивление как «сопротивление, блокирующее, препятствующее или ограничивающее ток, протекающий по цепи». Примеры сопротивления в цепи — электрическая лампочка или электрический прибор.

Если напряжение (давление воды) остается прежним, ток будет зависеть от сопротивления. Чем больше сопротивление, тем меньше ток. И наоборот, чем меньше сопротивление, тем больше ток. Это похоже на то, что чем больше вы перекрываете конец шланга, тем больше уменьшается поток воды, уменьшение сопротивления увеличивает поток воды.

Если сопротивление остается прежним, ток можно увеличивать или уменьшать, увеличивая или уменьшая напряжение (давление воды).

Георг Симон Ом

В 1827 году немец по имени Георг Симон Ом опубликовал книгу, описывающую взаимосвязь между напряжением, током и сопротивлением. Эта связь теперь известна как закон Ома. Закон Ома гласит, что ток прямо пропорционален напряжению и обратно пропорционален сопротивлению.

Математически это выглядит так:

Ток = напряжение / сопротивление

Символы и единицы измерения напряжения и тока такие же, как и в формуле мощности.Обратите внимание на стандартные термины сопротивления.

Имя Символ Единицы измерения
Текущий Я ампер или ампер (A)
Напряжение В вольт (В)
Сопротивление R Ом (Ом)

Следовательно, мы можем быстро записать эту формулу как:

I =
В / R

Популярный способ запомнить эту формулу и ее производные — это показанный здесь треугольник.Какую бы характеристику вы ни искали, прикройте ее пальцем, и формула, которая вам нужна, останется.

Пример 1. Чтобы найти напряжение в цепи, зная сопротивление и ток, коснитесь пальцем буквы «V», и формула будет I x R.

Пример 2: Чтобы найти ток в цепи, зная напряжение и сопротивление, приложите палец, чтобы закрыть «I», и формула: В / R

Пример 3: Чтобы найти сопротивление цепи, зная напряжение и ток, коснитесь пальцем буквы «R», и формула: В / I

Другими словами, формула в трех возможных формах:

В = I x R I =
В / R R = В / I

Используя эти формулы и следующую схему, можно увидеть взаимосвязь между сопротивлением и током, при условии, что напряжение остается прежним.

Пример 1: Учитывая, что напряжение 220 В, а сопротивление 110 Ом (Ом), мы можем рассчитать ток по формуле:

I = В / R = 220 / 110 = 2 А

Пример 2: То же напряжение (220 В), но теперь сопротивление составляет всего 2 Ом.

I = В / R = 220 / 2 = 110 А

То есть, чем больше сопротивление, тем меньше ток может протекать.Кроме того, чем меньше сопротивление, тем больший ток может протекать.

Может оказаться полезным снова использовать аналогию с водой. Если у вас есть водопроводная труба большого диаметра, то через нее может протекать много воды (тока), потому что «сопротивление» низкое (отсутствие засоров). Если есть закупорка в трубе (сопротивление), то (текущий) поток уменьшается. Чем больше засорение (чем выше сопротивление), тем меньше (ток) протекания.

Этот принцип проиллюстрирован в примерах выше.В примере 1 сопротивление 110 Ом позволяло проходить через цепь только 2 ампера. В примере 2 небольшое сопротивление (2 Ом) позволяло протекать по цепи большому току в 110 ампер.

Сопротивление — это все, что сопротивляется току, протекающему по цепи. Лампочка — это сопротивление, как утюг, кухонный комбайн или плита. В электронике есть небольшие компоненты, называемые резисторами, которые имеют фиксированное известное значение. Даже простой кабель имеет некоторое сопротивление, на самом деле причина использования толстых кабелей заключается в том, чтобы уменьшить сопротивление и позволить большему току проходить через него (например, пожарный шланг больше, чем садовый шланг, поэтому через него может течь больше воды) .

P.S .: мы только что рассмотрели основные принципы закона Ома, но шшш, никому не говори.

Если вы находите математику запутанной, попробуйте простой калькулятор закона Ома.

Серия

или параллельная

Часто вы не знаете, какое сопротивление в цепи, и вам не нужно это знать. Однако эти формулы по-прежнему очень полезны, но в основном для того, что на их основе выводится, а не для их непосредственного вычисления. Это длинный способ сказать: «пожалуйста, примите, что следующие концепции получены из приведенной выше формулы, и нам не нужно тратить время и силы на ее доказательство».

Концепция 1:

Рассмотрим схему из шести лампочек. Поскольку они подключаются друг за другом, говорят, что они подключены последовательно.

Интересно, что происходит с напряжением, током и сопротивлением.

Напряжение: «Входное напряжение» — это сумма напряжений на всех отдельных лампах. В этом примере индивидуальное напряжение на каждой лампе составляет 40 вольт, поэтому общее напряжение составляет 240 вольт.

Ток: Ток через каждый отдельный светильник такой же, как ток во всей цепи.Если ток, идущий в цепь, составляет 1 ампер, тогда ток через каждую отдельную лампу также составляет 1 ампер.

Сопротивление: Общее сопротивление равно сумме всех отдельных «последовательных» сопротивлений.

На практике этот тип цепи редко используется в доме. Это связано с тем, что если в дом поступает 240 вольт, а в доме есть шесть последовательно соединенных ламп, то каждый свет будет получать только одну шестую от этого напряжения, то есть 40 вольт. Тем не менее, этот тип схемы часто используется для гирлянды на елку. E.грамм. 20 лампочек по 12 вольт каждая равны 240 вольт. Проблема с этой схемой в том, что если горит одна лампочка, то погаснут и все остальные. Это связано с тем, что проводное соединение со следующим светом оборвано, поэтому цепь не завершена.

Часто используется последовательная цепь с батареями. Когда батареи включены последовательно, общее напряжение равно сумме всех отдельных напряжений, а общий ток, протекающий по всей цепи, такой же, как ток, протекающий через любую одну батарею.Например: четыре батареи на 1,5 В 500 мА, соединенные последовательно друг с другом, составляют батарею на 6 В (4 x 1,5) при 500 мА.

КОНЦЕПЦИЯ 2:

Рассмотрим схему из шести лампочек. Поскольку они соединены друг с другом или параллельно друг другу, говорят, что они соединены параллельно.

То, что происходит с напряжением, током и сопротивлением в цепи этого типа, отличается от того, что происходит в последовательной цепи.

Напряжение: напряжение на каждом индикаторе такое же, как «напряжение на входе».Если в цепь поступает 220 вольт, значит, 220 вольт на каждой отдельной лампе.

Ток: Ток, протекающий в цепи, представляет собой сумму токов в каждом отдельном источнике света. Если ток через каждую лампу составляет 1 ампер, то общий ток, поступающий в эту цепь, будет 6 ампер (6 x 1 ампер).

Сопротивление: общее сопротивление рассчитывается по сложной формуле, которую нам не нужно изучать на данном этапе. Достаточно сказать, что общее сопротивление меньше наименьшего индивидуального сопротивления.Вот формула для тех, кому действительно нужно знать:

На практике этот тип схемы используется в жилых домах. например Если у вас в доме 220 вольт, то все светильники и розетки (розетки) тоже 220 вольт, потому что они соединены параллельно.

Батареи также можно подключать параллельно. Выходное напряжение двух параллельно включенных батарей такое же, как у одной из батарей, но текущая емкость удваивается. Например: две автомобильные батареи на 12 вольт на 100 ампер / час, подключенные параллельно, составляют эквивалент 12-вольтовой батареи на 200 ампер / час.Те же две батареи, соединенные последовательно, будут эквивалентны батарее на 24 вольта и 100 ампер / час.

Собираем все вместе

Некоторые люди в восторге от того, как формулу закона Ома можно подставить в нашу формулу Силы для создания новых формул. Те из вас, кто любит играть с формулами, могут понять, как это делается; в остальном просто согласитесь, что эта таблица показывает различные комбинации этих формул.

В этой таблице показаны все возможные комбинации с использованием двух изученных нами основных формул.Попробуйте использовать калькулятор и следующие значения, чтобы проверить это:

В = 12 В I = 2 А R = 6 Ом P = 24 Вт

Вы можете распечатать таблицу и прикрепить ее к футляру мультиметра или стене мастерской для использования в будущем.

Простой калькулятор для вычисления всех этих комбинаций доступен здесь.

Поздравляю, вы закончили читать эти первые две статьи об основах электротехники. Это было очень тяжело с теорией и всевозможными формулами, но теперь у вас есть основы для понимания большинства электрических и электронных принципов.

Видеоурок: Закон Ома | Нагва

Стенограмма видео

В этом видео мы собираемся говоря о законе Ома. Этот закон был разработан немецким физик Джордж Ом в 1800-х годах. Как мы увидим, этот закон должен с электрическими цепями. И, в частности, соединяет ток, напряжение и сопротивление в этих цепях.

Во времена Ома эти концепции напряжение, ток и сопротивление в цепях были известны.Но им было не очень хорошо понял. Итак, Ом разработал эксперимент, чтобы лучше понять их. Он установил простой электрический цепь, включающая источник напряжения, в те времена называлась гальванической батареей. А потом он собрал набор проводников разной длины, толщины и даже из материала типы.

Использование одного из проводов для замкнув цепь, Ом приложил бы определенную разность потенциалов к схема.А потом установка гальванометра в этой схеме для измерения тока он считывал ток, который протекал через цепь в результате того, что этот конкретный проводник находится в ней под этим разность потенциалов.

После сбора данных, Ом изменит разность потенциалов в этой цепи, изменив высоту гальванической батареи и еще раз считайте ток, протекающий по этой цепи как результат.Затем, когда он закончил делать целую серию измерений для данного проводника, он переходил к другому из коллекции и сделайте то же самое, пробегите ряд потенциальных различия в цепи и записать ток, который будет течь через нее каждый раз.

После этого со всеми проводников, Ом собрал довольно много соответствующих данных по напряжению и току точки.Ом увидел, что эти точки могут быть нанесен на график. В своем эксперименте независимый переменной было напряжение, приложенное к цепи. Зависимая переменная была ток, который в результате будет протекать по цепи.

Ом обнаружил, что когда чертил все эти точки данных, рассматривая каждого проводника отдельно, Ом обнаружил, что если бы он нарисовал линия наилучшего соответствия через точки данных от каждого отдельного проводника, что-то интересное выделилось.В каждом случае линия наилучшего соответствия действительно была линией с постоянным наклоном. И эта линия прошла через источник. Ома заметил, что это подразумевало очень определенную взаимосвязь между током в этой цепи и напряжение на нем.

Эти прямые отношения для каждого проводника, который он тестировал, подразумевал, что ток в этой цепи был прямо пропорционально напряжению на нем.Это означает, что если бы мы удвоить напряжение в цепи для данного проводника, затем ток через этот проводник тоже удвоится. Мы можем увидеть это, если посмотрите на одну из этих линий, которая лучше всего подходит.

Выберем розовую линию для пристальный взгляд. Учитывая линейку наиболее подходящих для этого конкретного проводника, если мы переместим из начала координат две отметки вдоль горизонтальная ось, то это означает конкретное значение разности потенциалов по цепи.Мы не знаем, что это за ценность навскидку. Но мы знаем, что если проследим это до линии, наиболее подходящей для розового проводника, тогда он соответствует току через этот проводник две отметки вверх по вертикальной оси. Итак, две отметки на ось напряжения, сколько бы вольт она ни была, соответствует двум отметкам на шкале текущая ось, независимо от текущего значения.

А теперь допустим, что мы удвоили напряжение, приложенное к этому конкретному проводнику.Выходим четыре галочки. Если мы затем проследим от этой точки пока мы не дойдем до розовой линии наилучшего соответствия, а затем проследим до соответствующего current, мы видим, что теперь это четыре метки вверх по оси от начала координат. Другими словами, мы удвоили напряжение, приложенное к этому проводнику. И в результате мы удвоил ток через него. Вот что значит текущий прямо пропорциональна напряжению.

А на самом деле мы можем взять это отношения — 𝐼 прямо пропорционально 𝑉 — и мы можем записать это по-другому способ. Математически эквивалентный способ напишите это, чтобы сказать, что 𝐼 равно некоторой константе — мы назовем это 𝐶 — умноженное на напряжение. А здесь эта константа 𝐶 равна называется константой пропорциональности.

Итак, мы сказали, что закон Ома связывает эти понятия напряжения, тока и сопротивления в электрическом схема.Ом видел, что для каждого из проводников, которые он тестировал, при условии, что линия наилучшим образом соответствует точкам данных из этот проводник действительно образовывал линию, значит, эта постоянная пропорциональность была равна единице по сопротивлению проводника. То есть наклон каждого из этих линий для отдельных проводников равняется единице по сопротивлению дирижер.

Важно понимать, что наклон, который мы могли бы обозначить, используя строчную букву 𝑚, подразумевает другое значение сопротивления для каждого конкретного проводника.Они не все одинаковые сопротивление. Но учитывая этот конкретный резистор значение для проводника, это сопротивление остается неизменным независимо от того, сколько ток пропускаем через проводник. Вот что увидел Ом. Так что это действительно секрет Закон Ома, что это сопротивление, записанное в этом уравнении, является постоянной величиной. независимо от того, какое напряжение мы прикладываем к цепи и, следовательно, сколько ток проходит через него.

Теперь мы можем задаться вопросом, всегда ли это случай? То есть всегда ли правда, что независимо от материала, из которого сделан наш резистор, когда мы наносим на график данные точек из этого материала на кривой 𝐼 против, мы получим прямую линию? Короткий ответ на это — нет. Не все материалы ведут себя как те, которые мы видим здесь. Чтобы увидеть, как это может выглядеть, давайте освободим немного места на нашем графике.

Представьте, что мы находим еще один проводник из другого материала и провести эксперимент по записи напряжение и ток на нем.И представьте себе, что когда мы Нанесите эти точки данных на график, и мы обнаружим такую ​​взаимосвязь. И когда мы поместим это в линию лучше всего подходит, мы видим, что эта линия будет иметь изгиб. У него не будет постоянного наклона.

Напомним, что мы говорили, что наклон этой линии, которую мы видели ранее, золотой линии, равен единице над сопротивление этого проводника. И что критично, поскольку наклон эта линия везде одинакова, это означает, что сопротивление проводника равно везде тоже самое.Это константа. Такие материалы, которые имеют постоянное значение сопротивления независимо от того, какой ток проходит через них иметь определенное имя. Их называют омическими материалами.

Теперь интересно, в этом другом В данном случае наклон линии по-прежнему равен единице над сопротивлением. Но ясно, что для этой строки наклон не постоянный на всем протяжении. Он начинается довольно ровно, а затем увеличивается, пока не станет почти вертикальной линией вверху.Поскольку наклон меняется, это означает изменяется и сопротивление этого проводника. И это сопротивление зависит, следовательно, по току, проходящему через него.

Можно догадаться, что имя такой материал не омичен. То есть сопротивление материал не постоянный. Это действительно зависит от текущего пробегает материал. Когда дело доходит до омического и неомического материалы, если не указано иное, часто можно с уверенностью предположить, что материал омичен.Следовательно, следует Ома закон.

Говоря о законе Ома, мы можем прийти к наиболее знакомой форме этого закона, переписав это уравнение просто немного. Если мы умножим обе части уравнение их постоянным сопротивлением,, то этот член сокращается на Правая сторона. И мы видим, что 𝑅 раз 𝐼 равно равно 𝑉, или, что то же самое, равно 𝐼 раз.

И прежде чем двигаться дальше, давайте сделаем одно небольшое примечание об единицах измерения в этом выражении.В знак признания всех его кропотливой работы, единица сопротивления носит имя Джорджа Ома. Это называется ом. И это представлено греческим символ Ω.

Итак, при наличии определенного резистора мы сказал бы, что его сопротивление составляет пять Ом, 10 Ом или 100 Ом, или в любом другом случае может быть. Мы знаем, что единица измерения тока ампер и что единицей измерения напряжения является вольт. Итак, все это показывает нам, что один ом равно вольт, деленному на ампер.Или ом равен вольт на ампер. Зная все это, давай немного практики с использованием закона Ома на нескольких примерах.

У школьника есть резистор неизвестного назначения сопротивление. Она помещает резистор последовательно с источником переменной разности потенциалов. С помощью амперметра она измеряет ток через резистор при разных разностях потенциалов и графики ее результаты на графике, как показано на диаграмме.Какое сопротивление резистор?

Глядя на наш график, мы видим, что это график тока в амперах, протекающего через этот резистор, относительно напряжение в вольтах, проходящее через него. И на основании описания в постановку задачи, мы можем сделать небольшой набросок схемы, которая сгенерировала данные представлены здесь.

Допустим, это наш резистор неизвестного значения.Нам сказали, что этот резистор подключен к источнику с переменной разностью потенциалов, который также в этой цепи амперметр для измерения тока. Идея состоит в том, что мы используем это переменная подача разности потенциалов для подачи двух, четырех, шести и восьми вольт через этот резистор. А затем с помощью амперметра читаем соответствующие значения тока 0,4, 0,8, 1,2 и 1,6 ампер.

С этими значениями, нанесенными на графике, мы видим, что они соответствовали линии наилучшего соответствия, которая проходит непосредственно через все четыре точки, а также проходит через начало координат.Эта линия действительно является линией, имеет постоянный уклон. И именно этот наклон поможет ответим на этот вопрос, каково сопротивление нашего неизвестного резистора.

Чтобы узнать, как это сделать, вспомним закон. Этот закон говорит нам, что для резистор постоянного значения, это сопротивление, умноженное на текущий текущий через резистор равно напряжению на нем. В нашем случае мы хотим переставить это уравнение решить относительно 𝑅.И мы видим, что это равно разность потенциалов, деленная на ток. Нам не даны явные значения для разность потенциалов или ток. Но мы можем получить их из данных нанесенный на наш график.

Напомним, что эти точки данных основа для линии наилучшего соответствия, которая проходит через все они. Это означает, что для обеспечения напряжение и ток, которые нам нужно найти для сопротивления, 𝑅, мы можем выбрать из среди любой из четырех точек данных, представленных на этом графике.Фактически, мы могли выбирать из любого точки вдоль этой линии наилучшего соответствия, потому что так получилось, что она проходит идеально через все эти точки данных. Но чтобы упростить задачу, мы может также ограничить наш выбор этими четырьмя. Неважно, какой из четырех мы выбрали. Любой из них даст то же самое соотношение и, следовательно, тот же общий результат для сопротивления резистора.

И просто выбрать одну из точек тогда давайте выберем один на четыре вольта.Это напряжение соответствует ток, протекающий через резистор 0,8 ампера. Итак, чтобы решить проблему сопротивления резистора, мы разделим четыре вольта на 0,8 ампер. Когда мы это делаем, мы находим результат пять Ом, где Ом — единица сопротивления. Основываясь на нашем графике и законе Ома, мы находим сопротивление резистора равным пяти Ом.

А теперь рассмотрим еще один пример. закона Ома.

Резистор 10 Ом в цепи имеет разность потенциалов на нем составляет пять вольт.Какой ток через резистор?

Мы видим, что в этой задаче мы хотите связать эти три вещи: сопротивление, разность потенциалов и Текущий. Мы можем вспомнить математический отношения, которые действительно связывают все три, называемые законом Ома. Этот закон говорит нам, что если у нас есть резистор, значение которого не меняется в зависимости от протекающего через него тока, тогда, если мы умножим это сопротивление на ток, протекающий через него, мы получим разность потенциалов на нем.В этом случае безопасно предположим, что наш резистор 10 Ом действительно имеет постоянное значение сопротивления, что 10 Ом не будет зависеть от тока, протекающего через резистор.

Следовательно, мы можем смело применять это соотношение, что разность потенциалов на этом конкретном резисторе равна току через него, умноженному на его сопротивление. Как написано, это уравнение имеет решение для разности потенциалов.Но, конечно, мы не хотим найти разность потенциалов.

Хотим решить для тока. Для этого мы можем переставить это уравнение, поэтому оно читается как 𝐼, равно 𝑉, деленному на. И из нашей постановки задачи мы имеют значения 𝑉 и 𝑅, которые мы можем заменить. Мы работаем с 10-омным резистор. И напряжение на нем пять вольт. И когда мы рассчитываем это дробь, мы находим, что она равна 0.5 ампер. По закону Ома это ток, протекающий через этот резистор.

Давайте займемся моментом, чтобы Обобщите то, что мы узнали о законе Ома. В этом уроке мы увидели, что Закон Ома связывает ток, напряжение и сопротивление в электрических цепях. Когда это записано в виде уравнения, Закон Ома гласит, что для резистора с постоянным сопротивлением это значение резистора умноженный на текущий, он равен разности потенциалов через это.

Мы также видели, что в то время как очень многие компонентов в электрической цепи изготовлены из материалов, сопротивление которых, 𝑅, не зависит от протекающего через них тока, это не всегда кейс. Если сопротивление материала не зависит от того, сколько или как мало тока проходит через него, тогда это материал называется омическим материалом. С другой стороны, если значение сопротивления материала зависит от того, сколько тока проходит это, то этот материал называется неомным.

И мы видели, что, в общем, если только нам говорят иначе, обычно можно с уверенностью предположить, что данный материал и данный резистор омический. То есть это следует закону Ома. И, наконец, мы увидели, что единица измерения Сопротивление названо в честь первооткрывателя этого закона. Это называется ом. Ом обозначается с помощью Греческая буква Ω.

И мы увидели, что с точки зрения других единиц, ом равен вольт на ампер.Благодаря этому мы узнали о Закон Ома, который является одним из самых полезных, когда мы работаем с электрическими схемы.

Диаграмма

, уравнения и эксперимент — StudiousGuy

Сегодня невозможно представить мир без электричества. Из-за отсутствия электричества вся наша деятельность практически затруднялась бы. Как понимались основы электричества? С чего все началось? Такие вопросы могут быть для вас интригующими.Строительными блоками для управления и использования электричества являются напряжение, ток и сопротивление. Передачу энергии в электрических цепях невозможно обнаружить без помощи таких инструментов, как амперметр, вольтметр и т. Д. Джордж Саймон Ом был немецким физиком, который предложил связь между электрическим током и разностью потенциалов.

В этой статье мы делаем все возможное, чтобы сделать для вас кристально ясными основы напряжения, тока и сопротивления, а также их взаимосвязь с другими.

Закон Ома

Прежде чем мы подробно обсудим закон Ома, подумайте о постановке эксперимента.

  • Возьмите нихромовый провод, амперметр, вольтметр и четыре ячейки по 1,5 В каждая и установите схему, как показано.
  • Изначально используйте только одну ячейку в качестве источника энергии в цепи и запишите показания тока (I), отображаемые на амперметре, и разности потенциалов (V), отображаемые на вольтметре, по XY. XY здесь представляет собой нихромовую проволоку.Запишите эти показания в таблицу.
  • Теперь соедините две ячейки в цепь и снова запишите показания тока и разности потенциалов.
  • Теперь вы можете повторить тот же процесс, используя в схеме по отдельности три и четыре ячейки.
  • Следовательно, рассчитайте отношение V к I в каждом случае и постройте график между V и I.
  • Вы заметите, что график V-I будет прямой линией, проходящей через центр.
  • Вы также заметите, что значение V / I, полученное в каждом случае, будет примерно одинаковым.Следовательно, можно справедливо сказать, что V / I — постоянная функция.

Это было в 1827 году, когда Джордж Ом предложил соотношение между током (I), протекающим по металлическому проводу, и разностью потенциалов (V) на выводах металлического провода.

  • Ток (I), протекающий по концам металлического провода в электрической цепи, прямо пропорционален разности потенциалов (V) при условии, что температура постоянна. Это известно как закон Ома и обозначается как;
  • В приведенном выше уравнении R — постоянная величина, обозначаемая как сопротивление.Для данной металлической проволоки R постоянна при данной температуре. По закону Ома R = V / I.
  • Кроме того, согласно закону Ома, ток, протекающий по проводнику, обратно пропорционален сопротивлению проводника,

Теперь мы обсудим некоторые термины, которые используются при обсуждении закона Ома более подробно:

Электрический заряд

Электричество — это поток электронов. Электроны несут заряд, который обеспечивает энергию.Все источники света, такие как ламповые лампы, фонарики, лампочки и т. Д., Используют движение электронов для излучения света. Единица измерения электрического заряда в системе СИ — колумб (Кл).

Электрический ток

Количество электрического заряда, протекающего через область за единицу времени, называется электрическим током. Электрический ток — это скорость протекания электрических зарядов. Когда электричество было впервые обнаружено как явление, электроны еще не были известны. Следовательно, направление потока положительных зарядов было принято за направление электрического тока.

Напряжение

Количество потенциальной энергии между двумя точками в цепи определяется как напряжение. Также известная как разность потенциалов, это разница зарядов между любыми двумя точками. Он измеряется в вольтах (В). Разность потенциалов между любыми двумя точками в электрической цепи определяется как работа, совершаемая при перемещении единичного заряда из одной точки в другую, V = W / Q; где W = проделанная работа и Q = заряд. Разность потенциалов в 1 вольт между любыми двумя точками определяется как 1 Джоуль энергии, который передается на один столбец заряда.18 электронов), проходящие через точку в цепи за одну секунду. В уравнениях ток обозначается буквой «I».

Если «Q» — это чистый заряд, протекающий через проводник любого поперечного сечения за время «t»; тогда ток, протекающий через поперечное сечение проводника, представляется как;

Сопротивление

Сопротивление — это свойство проводника сопротивляться потоку электронов. Сопротивление проводника определяет величину тока.Единица измерения сопротивления в системе СИ — Ом (Ом). Сопротивление в 1 Ом определяется как сопротивление, которое при приложении напряжения 1 В позволит току в 1 А протекать по цепи.

Факторы, влияющие на сопротивление

Сопротивление жилы зависит от

  1. по длине,
  2. по площади поперечного сечения, а
  3. о характере материала.

Сопротивление проводника прямо пропорционально его длине (L) и обратно пропорционально площади его поперечного сечения (A),

Если мы объединим два вышеупомянутых уравнения, мы получим

В этих уравнениях ρ или rho — константа пропорциональности.ρ — удельное электрическое сопротивление материала проводника. Единица измерения удельного сопротивления в системе СИ — Ом · м. Удельное сопротивление — это свойство материала проводника. Это зависит от температуры.

  • Металлы и сплавы являются хорошими проводниками электричества; потому что они имеют низкое удельное сопротивление в диапазоне от 10 –8 Ом м до 10 –6 Ом м.
  • Резина и стекло плохо проводят электричество или изоляторы. Они имеют высокое удельное сопротивление в диапазоне от 10 12 до 10 17 Ом · м.

Общие сведения об электрических цепях

Если вам все еще трудно понять основы электрического заряда, тока, напряжения и сопротивления, вы можете рассмотреть обычную аналогию с резервуаром для воды. Количество воды в резервуаре представляет собой заряд, давление воды представляет собой напряжение, а поток воды представляет собой ток.

  • Вода = Заряд
  • Давление = Напряжение
  • Расход = Текущий

Резервуар для воды находится на определенной высоте над землей; шланг также присутствует в основании резервуара.

Напряжение представлено давлением на конце шланга. Поскольку вода внутри резервуара представляет собой заряд, следовательно, чем больше количество воды в резервуаре, тем выше будет заряд и больше будет давление на конце шланга. Здесь танк представляет собой батарею. Если в баке слито небольшое количество воды, давление на конце шланга падает. Это можно рассматривать как аналог состояния, при котором фонарик становится тусклым по мере разрядки батареек.

Теперь рассмотрим два танка; каждая со шлангом внизу. В обоих резервуарах одинаковое количество воды, но шланг одного из резервуаров уже, чем другой. Если вода начнет течь, скорость потока будет меньше для бака с более узким шлангом, чем для бака с более широким шлангом. Говоря с точки зрения электрических цепей, поток тока через более узкий шланг будет меньше по сравнению с протеканием тока через более широкий шланг.

Если мы хотим, чтобы из обоих резервуаров вытекало одинаковое количество воды, нам нужно увеличить количество воды, вытекающей из резервуара, с помощью более узкого шланга.В результате необходимо увеличить давление на конце более узкого шланга, чтобы протолкнуть больше воды через резервуар. Это похоже на увеличение тока, вызванное увеличением напряжения.

Добавить комментарий

Ваш адрес email не будет опубликован. Обязательные поля помечены *